Menu Close

prove-that-0-tanhx-x-3-sech-2-x-x-2-dx-7-pi-2-3-where-3-apery-s-constant-




Question Number 112369 by mathdave last updated on 07/Sep/20
prove that   ∫_0 ^∞ (((tanhx)/x^3 )−((sech^2 x)/x^2 ))dx=(7/π^2 )ζ(3)  where ζ(3)=apery′s constant
$${prove}\:{that}\: \\ $$$$\int_{\mathrm{0}} ^{\infty} \left(\frac{\mathrm{tanh}{x}}{{x}^{\mathrm{3}} }−\frac{\mathrm{sech}^{\mathrm{2}} {x}}{{x}^{\mathrm{2}} }\right){dx}=\frac{\mathrm{7}}{\pi^{\mathrm{2}} }\zeta\left(\mathrm{3}\right) \\ $$$${where}\:\zeta\left(\mathrm{3}\right)={apery}'{s}\:{constant} \\ $$

Leave a Reply

Your email address will not be published. Required fields are marked *